Recent content by SwordSmith

  1. S

    What do the eigennumbers of the Lorentz transformation represent?

    I am currently looking a bit into special relativity. Consider the matrix \Lambda=\left( \begin{array}{cc} \gamma & -\gamma \beta c \\ -\gamma \beta c & \gamma \end{array} \right) where \beta=\frac{v}{c},\quad \gamma=\frac{1}{\sqrt{1-\beta^2}} and c is the speed of light. Then, an observer...
  2. S

    What is Partial Gamma-production?

    So it describes the probability that a nuclear reaction will produce a prompt gamma with a certain energy or is it any prompt gamma in general? Example: 65Cu(p,n+γ(E=1200keV))65Zn - the partial γ-ray production cross-sections will thus describe the probability for this γ with this particular...
  3. S

    What is Partial Gamma-production?

    Hi I am currently looking at some literature for the production of different radioactive nuclei under the bombardment of protons on Copper. I found something called partial γ-ray production cross-sections and I am wondering what this means. I know that cross-sections generally describe the...
  4. S

    Derivative wrt Complex Conjugate

    I am not sure what the derivative with respect to a complex conjugate is and I have not been able to find it in any books. I assume I should use the chain rule somehow to figure this out: \frac{\partial z}{\partial z^*}, \quad z=x+iy Maybe you can do like this? \frac{\partial...
  5. S

    The Optimal Heat Sink for my Lousy Freezer

    For some reason, my freezer stops its pumps once in a while and it does not start again before I turn it off and on again. I would like to increase the time interval it takes for the freezer to heat up above 0°C where the food starts to decay faster. I therefore need a heat sink. The operating...
  6. S

    Inverse Fourier Transformation of a Fourier Transformation

    I am having problem with the inverse transformation of a Fourier transformed function which should give the function itself. Let f=f(x) and let f be Fourier transformable (whatever that implies) Let \tilde{f}(k)=∫^{\infty}_{-\infty}dx e^{-ikx}f(x) (1) then we should have...
Back
Top